Prove x13+x23+x33+x43 ≥ x1+x2+x3+x4 with x1x2x3x4=1

  • Thread starter asmani
  • Start date
In summary: I think you're confusing the induction step with the base case. The induction step shows that if the statement is true for some k, then it is true for k + 1. The base case proves that it is true for n = 1 (or some other starting point depending on the problem). The issue in the original problem was that n = 0 was not explicitly stated as a possible base case, so it needed to be considered separately.
  • #1
asmani
105
0
Suppose that x1,x2,x3,x4 are four positive real numbers such that x1x2x3x4=1.

Show that x13+x23+x33+x43 ≥ x1+x2+x3+x4.

How to solve this? (Not a homework!)

Any hint is appreciated.
 
Mathematics news on Phys.org
  • #2
Looks like it should be pretty easy with induction if you can prove the base case of:

if x1,x2 are positive real numbers such that x1x2=1 then x13 + x23≥ x1 + x2

For the base case you know that x1x2=1 ⇒ x2= 1/x1

i.e. show x3 + 1/x3 - x - 1/x ≥ 0, for x ≥ 0 which shouldn't be to bad with some calculus

At least that's the route I would go.
 
  • #3
MO Question?
 
  • #4
Use lagrange. It will pop right out.
 
  • #5
Thanks for the replies.

That's a part of a problem of a national mathematical olympiad. The full problem is as follows:

Suppose that x1,x2,x3,x4 are four positive real numbers such that x1x2x3x4=1. Show that

Cgeq%20%5Cmax%5Cleft%20%5C{%20%5Csum_{i=1}^{4}x_i,%5Csum_{i=1}^{4}%5Cfrac{1}{x_i}%5Cright%20%5C}.gif
 
  • #6
JonF said:
Looks like it should be pretty easy with induction if you can prove the base case of:

if x1,x2 are positive real numbers such that x1x2=1 then x13 + x23≥ x1 + x2

For the base case you know that x1x2=1 ⇒ x2= 1/x1

i.e. show x3 + 1/x3 - x - 1/x ≥ 0, for x ≥ 0 which shouldn't be to bad with some calculus

At least that's the route I would go.
Base case could just be n=1, i.e. one x. I guess you could also argue for n=0 too.
 
  • #7
Anonymous217 said:
Base case could just be n=1, i.e. one x. I guess you could also argue for n=0 too.
good point, that makes it even easier
 
  • #8
Anonymous217 said:
Base case could just be n=1, i.e. one x. I guess you could also argue for n=0 too.

Well, you got to watch out for which natural number corresponds to the base case when carrying out inductive proofs.

Check this out: We will show [itex]n^3 \leq n^2[/itex] for all n > 0 by induction. If n = 1, we get [itex]1^3 \leq 1^2[/itex] which is certainly true. Now assume [itex]k^3 \leq k^2[/itex]. Then

[tex](k + 1)^3 = k^3 + 3k^2 + 3k + 1 \leq k^2 + 3k^2 + 3k + 1 \leq k^2 + 2k + 1 = (k + 1)^2[/tex]

which was the desired result. But wait, [itex]2^3 = 8 > 4 = 2^2[/itex]. Contradiction? The problem comes from "choice" of base case.
 
  • #9
Analytical solution:

Let [tex]f(x_1,x_2,x_3,x_4) = x_1^3+x_2^3+x_3^3+x_4^3-(x_1+x_2+x_3+x_4)[/tex], and [tex]g(x_1,x_2,x_3,x_4) = x_1x_2x_3x_4-1[/tex].

we will find the minimum of f under the condition g = 0.

We find the solutions to the lagrange equations

[tex]\frac{\delta f}{\delta x_i} = \lambda \frac{\delta g}{\delta x_i}[/tex]

and g = 0, or

[tex]3x_i^2 -1= \lambda x_1x_2x_3 \Leftrightarrow 3x_i^3-x_i=\lambda[/tex], so [tex]3x_i^3-x_i = 3x_j^3-x_j[/tex], or [tex](x_i-x_j)(3(x_i^2+x_ix_j+x_j^2)-1) = 0[/tex]. Pick the largest one, say x_k. It will be larger or equal than 1. And thus [tex]3(x_k^2+x_ix_k+x_k^2)-1 \geq 2 > 0[/tex]. Hence [tex]x_i = x_k[/tex] for all i, so they are all equal. Hence x_i = 1 for all i.

Now this is the minimum on some compact set containing (1,1,1,1).

The set determined by [tex]x_i > 0[/tex], and [tex]x_1x_2x_3x_4 = 1[/tex], and [tex]x_1+x_2+x_3+x_4 <=5[/tex] contains (1,1,1,1) and is compact.

Furthermore, by the general QM-AM [tex]\sqrt[3]{\frac{x_1^3+x_2^3+x_3^3+x_4^3}{4}} \geq \frac{x_1+x_2+x_3+x_4}{4}[/tex]

So [tex]x_1^3+x_2^3+x_3^3+x_4^3 \geq \frac{(x_1+x_2+x_3+x_4)^2}{16} (x_1+x_2+x_3+x_4)[/tex]

Hence outside our set [tex]f(x_1,x_2,x_3,x_4) > 0[/tex]. So (1,1,1,1) is actually the value for which f is minimized, and the value is 0. Hence [tex]x_1^3+x_2^3+x_2^3+x_4^3 \geq x_1+x_2+x_3+x_4[/tex].

(EDIT: I assumed we were working in the subset x_i > 0 originally, so just assume I said that before I did :) )
 
Last edited:
  • #10
Dr. Seafood said:
Well, you got to watch out for which natural number corresponds to the base case when carrying out inductive proofs.

Check this out: We will show [itex]n^3 \leq n^2[/itex] for all n > 0 by induction. If n = 1, we get [itex]1^3 \leq 1^2[/itex] which is certainly true. Now assume [itex]k^3 \leq k^2[/itex]. Then

[tex](k + 1)^3 = k^3 + 3k^2 + 3k + 1 \leq k^2 + 3k^2 + 3k + 1 \leq k^2 + 2k + 1 = (k + 1)^2[/tex]

which was the desired result. But wait, [itex]2^3 = 8 > 4 = 2^2[/itex]. Contradiction? The problem comes from "choice" of base case.

k^2 + 3k^2 + 3k + 1 \leq k^2 + 2k + 1 that does follow from the induction hypothesis and is certainly not true.
 
  • #11
That's what I said.
 
  • #12
Dr. Seafood said:
That's what I said.

No, you derived those inequalities in your post, but it's wrong. How does it have anything to do with the base case?
 

Related to Prove x13+x23+x33+x43 ≥ x1+x2+x3+x4 with x1x2x3x4=1

1. How do you prove the inequality x13+x23+x33+x43 ≥ x1+x2+x3+x4 with x1x2x3x4=1?

To prove this inequality, we can use the AM-GM inequality which states that for any set of positive real numbers, the arithmetic mean is greater than or equal to the geometric mean. In this case, we can rewrite the given expression as x1+x1+x1+x1+x2+x2+x2+x2+x3+x3+x3+x3+x4+x4+x4+x4 ≥ 4√(x1x2x3x4) = 4√1 = 4. Therefore, x13+x23+x33+x43 ≥ x1+x2+x3+x4.

2. What is the significance of x1x2x3x4=1 in the inequality?

The condition x1x2x3x4=1 is important because it allows us to use the AM-GM inequality. This condition ensures that the numbers x1, x2, x3, x4 are all positive and that their product is equal to 1.

3. Can you provide an example to illustrate the given inequality?

Sure, let's say x1 = 1, x2 = 2, x3 = 3, and x4 = 1/6. In this case, x1x2x3x4 = 1 and x13+x23+x33+x43 = 1+8+27+1/216 = 36.0231. On the other hand, x1+x2+x3+x4 = 6.5. Therefore, x13+x23+x33+x43 ≥ x1+x2+x3+x4.

4. Is the given inequality always true for any set of positive real numbers?

Yes, the inequality x13+x23+x33+x43 ≥ x1+x2+x3+x4 with x1x2x3x4=1 is always true for any set of positive real numbers. This is because of the AM-GM inequality which holds true for any set of positive real numbers.

5. Can this inequality be applied to any other expressions involving products and sums?

Yes, the AM-GM inequality can be applied to any expression involving products and sums of positive real numbers. It is a useful tool in proving various mathematical inequalities and has many applications in different fields of science and engineering.

Similar threads

  • Engineering and Comp Sci Homework Help
Replies
14
Views
2K
  • Precalculus Mathematics Homework Help
Replies
2
Views
1K
  • Calculus and Beyond Homework Help
Replies
9
Views
2K
Replies
22
Views
5K
  • Precalculus Mathematics Homework Help
Replies
6
Views
9K
Replies
5
Views
3K
  • Precalculus Mathematics Homework Help
Replies
3
Views
1K
  • Calculus and Beyond Homework Help
Replies
8
Views
3K
  • Engineering and Comp Sci Homework Help
Replies
8
Views
3K
  • Calculus and Beyond Homework Help
Replies
2
Views
1K
Back
Top